Difference between revisions of "2010 AMC 12A Problems/Problem 4"

(Created page with '== Problem 4 == If <math>x<0</math>, then which of the following must be positive? <math>\textbf{(A)}\ \frac{x}{\left|x\right|} \qquad \textbf{(B)}\ -x^2 \qquad \textbf{(C)}\ -2…')
(No difference)

Revision as of 13:55, 10 February 2010

Problem 4

If $x<0$, then which of the following must be positive?

$\textbf{(A)}\ \frac{x}{\left|x\right|} \qquad \textbf{(B)}\ -x^2 \qquad \textbf{(C)}\ -2^x \qquad \textbf{(D)}\ -x^{-1} \qquad \textbf{(E)}\ \sqrt[3]{x}$

Solution